Functional Equation $f\Big(y\,\big(f(x)\big)^2\Big)=x^3\,f(xy)$

functional-equations

Let $f:\mathbb{Q}^+\to \mathbb{Q}^+$ satisfies the following equation $$ f\Big(y\,\big(f(x)\big)^2\Big)=x^3\,f(xy)$$ for all positive rationals $x,y$. Show that $f(x)=\dfrac1x$ for all $x\in\mathbb{Q}^+$. Show that there are infinitely many functions $f:\mathbb{R}\to \mathbb{R}$ satisfying the above equation.

I proved that $f(1)=1$ and if $f(2)=a$ then $f(a^{2n}/8^n)=8^n$ for all integers $n$ and I get no approach here on. Can anyone help?

Best Answer

In this answer, we assume that $f:\mathbb{Q}_{>0}\to\mathbb{Q}_{>0}$. Let $P(x,y)$ denote the condition that $$f\Big(y\,\big(f(x)\big)^2\Big)=x^3\,f(xy)\,.$$ First, we shall prove that $f$ is injective. If $x$ and $y$ are positive rational numbers such that $f(x)=f(y)$, then $$x^3\,f(x)\overset{P(x,1)}{=\!=\!=}f\Big(\big(f(x)\big)^2\Big)=f\Big(\big(f(y)\big)^2\Big)\overset{P(y,1)}{=\!=\!=}y^3\,f(y)\,.$$ As $f(x)=f(y)>0$, we get $x^3=y^3$, whence $x=y$.

Observe that $$\begin{align}f\Big(\big(f(x)\big)^2\,\big(f(y)\big)^2\Big)&\overset{P\Big(y,\big(f(x)\big)^2\Big)}{=\!=\!=\!=\!=\!=\!=}y^3\,f\Big(y\,\big(f(x)\big)^2\Big) \overset{P(x,y)}{=\!=\!=}y^3\,\Big(x^3\,f(xy)\Big) \\&=(xy)^3\,f(xy) \overset{P(xy,1)}{=\!=\!=\!=}f\Big(\big(f(xy)\big)^2\Big)\,.\end{align}$$ Because $f$ is injective, we get $$\big(f(x)\big)^2\,\big(f(y)\big)^2=\big(f(xy)\big)^2\,.$$ This shows that $f$ is multiplicative, namely, $$f(xy)=f(x)\,f(y)\text{ for all }x,y\in\mathbb{Q}_{>0}\,.$$

Thus, from $P(x,1)$, we get $$\Big(f\big(f(x)\big)\Big)^2=f\left(\big(f(x)\big)^2\right)=x^3\,f(x)\text{ for every }x\in\mathbb{Q}_{>0}\,.$$ Define $g(x):=x\,f(x)$ for each $x\in\mathbb{Q}_{>0}$. Note that $$\begin{align}\Big(f\big(g(x)\big)\Big)^2&=f\left(x^2\,\big(f(x)\big)^2\right)=\big(f(x)\big)^2\,\Big(f\big(f(x)\big)\Big)^2 \\&=\big(f(x)\big)^2\,\big(x^3\,f(x)\big)=\big(x\,f(x)\big)^3=\big(g(x)\big)^3\,.\end{align}$$ Consequently, $$\Big(g\big(g(x)\big)\Big)^2=\Big(g(x)\,f\big(g(x)\big)\Big)^2=\big(g(x)\big)^2\,\big(g(x)\big)^3=\big(g(x)\big)^5\,.\tag{*}$$

We claim that $g\equiv 1$. To prove this, we define the height $\text{ht}(r)$ of a rational number $r>0$ to be the largest integer $\beta\geq 0$ such that $2^\beta$ divides $\alpha_1,\alpha_2,\ldots,\alpha_k$, if $$r=p_1^{\alpha_1}\,p_2^{\alpha_2}\,\cdots\,p_k^{\alpha_k}\,,$$ where $p_1,p_2,\ldots,p_k$ are pairwise distinct prime natural numbers and $\alpha_1,\alpha_2,\ldots,\alpha_k\in\mathbb{Z}$. We set $\text{ht}(1)$ to be $\infty$. Clearly, $\text{ht}\big(f(x)\big)\geq \text{ht}(x)$ due to multiplicativity of $f$.

To finish the proof, we assume on the contrary that $g(x)\neq 1$ for some $x\in\mathbb{Q}_{>0}$. Let $u\in\mathbb{Q}_{>0}$ be such that $g(u)\neq 1$ and that $\text{ht}\big(g(u)\big)$ is minimized. By (*), we see that $v:=g(u)$ satisfies $$\text{ht}\big(g(v)\big)=\text{ht}\big(g(u)\big)-1\,.$$ This contradicts the choice of $u$. Therefore, $u$ cannot exist, and the claim follows. This implies that $$f(x)=\frac1x\text{ for all }x\in\mathbb{Q}_{>0}\,.$$